plz help with this will mark as brinliest

Plz Help With This Will Mark As Brinliest

Answers

Answer 1

Answer:

it’s c

Step-by-step explanation:


Related Questions

Perpendicular Bisector Theorem

Answers

A perpendicular bisector of a line segment is a line which divides the line segment into two equal parts. So here, PS = PQ

=》 PS = 3a

=》 PQ = a + 16

PS = PQ

3a = a + 16

3a - a = 16

2a = 16

a = 16/2

a = 8.

_______

RainbowSalt2222 ☔

Hello there can you please help y

Answers

Answer:

5x + 28 = 7x - 4

2x = -32

x = -16

Can someone help please

Answers

Answer:

0.3 miles per hour

Step-by-step explanation:

Speed is distance divided by time so your equation is 45 divided by 150. 45 divided by 60 plus 60 plus 30 = 150 45 divided by 150= 0.3

Suppose you invest 18,400.00 into an account earning an interest rate of 2.991% compounded continuously for 2 year(s) and thereafter earning an interest rate of 4.192%compounded yearly. How much money is in the account after 9 years?

Answers

[tex]~~~~~~ \stackrel{\textit{for the first 2 years}}{\textit{Continuously Compounding Interest Earned Amount}} \\\\ A=Pe^{rt}\qquad \begin{cases} A=\textit{accumulated amount}\\ P=\textit{original amount deposited}\dotfill & \$18400\\ r=rate\to 2.991\%\to \frac{2.991}{100}\dotfill &0.02991\\ t=years\dotfill &2 \end{cases} \\\\\\ A = 18400e^{0.02991\cdot 2}\implies A = 18400e^{0.05982}\implies A\approx 19534.276[/tex]

now let's grab that amount and invest it for the remaining 7 years at 4.192% compounded.

[tex]~~~~~~ \stackrel{\textit{for the last 7 years}}{\textit{Compound Interest Earned Amount}}\\\\A=P\left(1+\frac{r}{n}\right)^{nt}\quad\begin{cases}A=\textit{accumulated amount}\\P=\textit{original amount deposited}\dotfill &\$19534.276\\r=rate\to 4.192\%\to \frac{4.192}{100}\dotfill &0.04192\\n=\begin{array}{llll}\textit{times it compounds per year}\\\textit{yearly meaning once}\end{array}\dotfill &\\t=years\dotfill &7\end{cases}[/tex]

[tex]A=19534.276\left(1+\frac{0.04192}{1} \right)^{1\cdot 7}\implies \boxed{A\approx 26039.82}[/tex]

Find the differential of the function.
u = 3x4 + 4

Answers

Answer:

U(x)= 3+4

Step-by-step explanation:

I simplified this equation into function notation form, then simplified the 12 and 4 with the common factor of 4.

Hope this helped and have a great day,

Ginny

Answer:

14

Step-by-step explanation:

you could solve this three different ways

1. 4*3 is 12  so add 4 = 16

2. 4 squared because 4*4 = 16

3. draw 4 groups of four to get 16

8 2/3 and 10 23/32 in its simplest form

Answers

8 2/3 is 26/3 and 10 23/32 is 342/32

A machine packs 180 boxes of cereal in a half-hour. It takes 55 minutes to pack 330 boxes. True or False?

Answers

Answer:

True

Step-by-step explanation:

If you divide 180 by 30 you will get 6. then 6 times 5 is 30. If you add 30 to 330 you will get 360. If you take 180 boxes for 30minutes and double. You get 360 boxes in 60minutes. If 5 minutes is 30 boxes. Subtract 30 from 360 and you get 330 in 55 minutes.

Eli is deciding between two different movie streaming sites to subscribe to. Plan A costs $14 per month plus $2 per movie watched. Plan B costs $10 per month plus $3 per movie watched. Let A represent the monthly cost of Plan A if Eli watches xx per month, and let B represent the monthly cost of Plan B if Eli watches xx movies per month. Graph each function and determine the interval of movies watched, x,x, for which Plan A is cheaper than Plan B.

put it on a graph

Answers

Hope this helps I really tried

By analyzing the graph, we can see that plan B is cheaper for the interval [0, 4), while plan A is better if you look 4 or more movies per month.

How to get the cost equations?

We know that plan A costs $14 per month plus $2 per movie, so if you watch x movies, the cost is:

C₁(x) = $14 + $2*x

Plan B costs $10 per month plus $3 per movie, so if you watch x movies, the cost is:

C₂(x) = $10 + $3*x

Now to compare the costs, you can see the graph at the end. Where C₂(x) is the blue line, and C₁(x) is the green line.

In the graph, you can see that between x = 0 and x = 4, the blue line is below the green one. So, in the interval [0, 4) plan B is cheaper.

While for x > 4, the green line is below the blue one, so for x > 4 you should choose plan A.

If you want to learn more about linear equations, you can read:

https://brainly.com/question/4074386

Lindsey went to best buy and bought an MP3 Player. If the MP3 Player costs $78.99 and the sales tax was 6.5%, what was her total bill?

Answers

Answer:

$84.12

Step-by-step explanation:

78.99 x 0.065 = 5.13

78.99 + 5.13 = 84.12

Find the inequality represented by the graph

Answers

Answer:

y < 3/4 x

(We don't put another number at the end, if the y-intercept is 0, btw.)

Step-by-step explanation:

We know that the line has a positive slope, because it's going upwards to the right.

(We can use rise/run to determine the exact slope: Just go up 3 units and to the right 4 units, starting from the first point. Now we know the slope is 3/4).

Further more we know that the y-intercept is 0, because that's the point in which the line crosses the y-axis.

We also know that since the line is dotted, the sign will be either < or >.

And the shaded area below indictes that y is lesser than, meaning we use this sign <.

Hope this helps you :)

Luna is competing in a 200 m race.
She runs at a constant speed of 5.4 m/s for the first 50 m, then 4.4 m/s for 10 seconds.
The remainder of the race takes her 29 seconds to complete.
What is Luna's average speed for the entire race to 1 dp?

Answers

Answer:

24

Step-by-step explanation:

Answer:

24 in the answer

Step-by-step explanation:

If XZ=8x-18 and RZ = 2x+5 find XR

Answers

Answer:

6x+23

Step-by-step explanation:

8x-18+2x+23

8x-2x+5+18

6x+23

Answer:

XR=19

Step-by-step explanation:

Since the entire thing of XZ equals 8x-18 and one side of the line equals 2x+5 we will need another half to get the entire thing.

8x-18=4x+10

4x=28

x=7

Now we have to plug the x in

2(7)+5

=19

Harry is paid £8.60 per hour for the first 30 hours he works each week. After 30 hours he is paid 1 and a half times the hourly rate.

Last week Harry worked 33 hours. He was also paid a bonus of 1/10 of his earnings

Calculate how much in total Harry was paid last week

Answers

Answer:

1404.81

Step-by-step explanation:

1277.1 +127.71 =  i read it wrong the first time sorry

The total amount paid to harry in past week, is 299.28.

What is amount?

Amount is the sum or the total value.

For example, if A purchase 2 kg rice at the price of 40 rs/kg, the amount will be 40×2= 80 rs.

Given that,

The amount paid to Harry for 1 hour = $8.60

Since, Harry worked for 33 hours,

So, The amount paid to Harry for 33 hours = 8.60×33=283.8

Harry gets half times more amount than hourly rate and also gets bonus for last 3 days

So for last 3 days the extra amount paid to Harry

= 3×1/2×8.60+3×1/10×8.60

= 12.9+2.58

=15.48

So the total amount paid to harry = 283.8+15.48=299.28

To know more about amount on :

https://brainly.com/question/28244711

#SPJ2

Which one of the following statements about fractions is correct?
Question 9 options:

1/3 is an improper fraction.

9/7 is a proper fraction.

5/16 is a mixed number.

4/4 is equal to 1.

Answers

Answer:

4/4 is equal to 1 is the only one that's correct

Step-by-step explanation:

Answer:

4/4 and 1/3 are the correct statements

PLEASE HELP 10 POINTS✨

Write an equation for each problem. Use the variable M.

18) Five students are absent from math class today. If there are 22 students in math class today, how many students would there be if none were absent? =

19) Juan pays $22.05 more than Kim each month for his health club membership. Juan pays $59.95 each month. How much does Kim pay each month? =

Answers

Answer: 18)  there are 27 students

i am new to this so i am sorry if i got this wrong

Step-by-step explan If there are 22 students add 22+5 and the anwser is 27

x + ( -22 ) = 31
x = ?​

Answers

Answer :

x + ( -22 ) = 31

x = ?

Answer :

x = 31 - ( -22 )

x = 31 + 22 =

x = 53

x = 53 ✔️

Answer:

52

Step-by-step explanation:

x + (-21)=31

x-21=31

x=31+21

x=52

1.25 is closer to 1.04 or not ?
plz heelp

Answers

Answer:

Yes 1.25 is closer to 1.04

Step-by-step explanation:

Hope this helps!

Answer:

1.25 is not closer to 1.04 because it's like, 1 dollar and 4 cents; compared to 1 dollar and 25 cents. Obviously 25 cents is more than 4 cents.

1.25 rounded:

1.30

1.04 rounded:

1.00

A hose fills a hot tub at a rate of 4.86 gallons per minute. How many hours will it take to fill a ​267 gallon hot​ tub?

Answers

55 minutes i think. I did 267 divided by 4.86 and it was 54.9 and i rounded it

Steve bicycles 3 mi to school everyday at 15 mph. If school starts at 8:30 am and he needs 15 min to lock his bike and get to his desk then what time should he leave his house

Answers

Answer:

8:03

Step-by-step explanation:

time=speedxdistance

t=1/15x3

t=3/15hr

t=.2hr

t=12 mins

plus 15 mins to lock up bike, 27 mins

27 mins before 8:30 is 8:03

pls mark branliest

HELP WITH THIS PLS :)0000

Answers

Answer:

5 to 1 power

Step-by-step explanation:

Answer:

5^(15) or 5 to the 15th power

Step-by-step explanation:

When simplified, both equal to 305175..

The mean of 4 numbers is 40, what is the sum of the 4 numbers

Answers

Answer:

the sum of 4 numbers is 160

Step-by-step explanation:

Let 4 numbers be a, b, c, d

Mean of those 4 numbers is: (a+b+c+d) : 4 = 40

So the sum of 4 numbers is: (a+b+c+d) = 40 x 4 = 160

=> 160 is the sum of 4 numbers

g(x)=x^2-6/3x+10 g(4)=?

Answers

Answer:

g(4) = [tex]\frac{5}{11}[/tex]

Step-by-step explanation:

Substitute x = 4 into g(x) , that is

g(4) = [tex]\frac{4^2-6}{3(4)+10}[/tex] = [tex]\frac{16-6}{12+10}[/tex] = [tex]\frac{10}{22}[/tex] = [tex]\frac{5}{11}[/tex]

>> Answer

_______

[tex] \: [/tex]

[tex] \sf{g(x) = \frac{ {x}^{2} - 6 }{3x + 10} }[/tex]

[tex] \sf{g(4) = \frac{ {4}^{2} - 6 }{3(4) + 10}} [/tex]

[tex] \sf{g(4) = \frac{16 - 6}{12 + 10}} [/tex]

[tex] \sf{g(4) = \frac{10}{22}} [/tex]

[tex] \sf{g(4) = \bold{\frac{5}{11}} }[/tex]

What is the quotient of 8×10 to the 20/4÷2×10 to the 18th in scientific notation

Answers

Step-by-step explanation:

[tex] = \frac{8 \times {10}^{24} }{ 2\times {10}^{18} } [/tex]

[tex] = \frac{8}{2} \times {10}^{24 - 18} [/tex]

[tex] = 4 \times {10}^{6} [/tex]

[tex] \: [/tex]

[tex] \frac{8 \times {10}^{24} }{2 \times {10}^{18} } = 4 \times {10}^{6} [/tex]

Answer:

[tex] \sf \frac{8 \times {10}^{24} }{2 \times {10}^{18} } = ... \times {10}^{?} [/tex]

[tex] \sf (8 \div 2) \times {10}^{(24 - 18)} =... \times {10}^{?} [/tex]

[tex] \sf4 \times {10}^{6} = ... \times {10}^{?} [/tex]

[tex]↪4 \times {10}^{6} [/tex]

La función de costos para una compañía es C=125q + 13500 y su punto de equilibrio es q=270. Plantear la función de Utilidad

Answers

Based on the information given, the total cost incurred based on the equation given as C= 125q + 13500 will be 47250.

From the information given, the cost function is given as C=125q + 13500 while the value of q is given as 270. Therefore, on order to find the cost, we'll slot in the value of q into the equation given. This will be:

C=125q + 13500

C = 125(270) + 13500

C = 33750 + 13500

C = 47250

In conclusion, the cost is 47250.

Read related link on:

https://brainly.com/question/25441033


Jasmine travels 732 miles at an average speed of 65 mi/hr. How long will it take her to reach her destination? Round your answer to
the hundredths place.

Answers

Answer:

11.26 hr

Step-by-step explanation:

Use the following equation:

Time  = distance / speedt = 732 / 65 = 11.26 (rounded)

Answer:11.26 hours

Step-by-step explanation:

QUESTION
arning Page
The length of a rectangle is 3 m less than twice the width, and the area of the rectangle is 65 m². Find the dimensions of the rectangle.

Answers

Answer:

width: 6.5 mlength: 10 m

Step-by-step explanation:

Let w represent the width of the rectangle in meters. Then (2w-3) is the length. The area is the product of length and width:

  A = LW

  65 = (2w-3)(w)

  2w² -3w -65 = 0

  (2w -13)(w +5) = 0 . . . . . . . factor

  w = 6.5 . . . . . . . . the positive solution

  2(6.5) -3 = 10 . . . the length of the rectangle

The width of the rectangle is 6.5 meters; the length is 10 meters.

(2a-1/3) / b/15 when a=-3/5 and b=-6.75

Answers

Answer:

4.148148149

Step-by-step explanation:

that's the answer

Step-by-step explanation:

2a-1/3 / b/15

[tex](2 \times \frac{ - 3}{5 } ) \div \frac{ - 6.75}{15} \\ = \frac{ - 6}{5} \times \frac{15}{ - 6.75} \\ = \frac{18}{6.75} [/tex]

If the function f (x) is defined by f(x) =x over 2 minus 6

Answers

Depending on how it is written, it could be either of these following solutions:

f(x) = x/2 - 6     or     f(x) = x/(2-6)

For the first, we solve for x by replacing f(x) with 0

x/2 - 6 = 0

x/2 = 6

x = 3

For the second, do the same

x/(2-6) = 0

x/-4 = 0

x = 0

Theresa Morgan, a chemist, has a 30% hydrochloric acid solution and a 55% hydrochloric acid solution. How many liters of each should she mix to get 437.5 liters of a hydrochloric acid solution with a 45% acid concentration?

Answers

Answer:

Step-by-step explanation:

30F + 55S = 45(437.5)

30F + 55S = 19687.5

30F = 19687.5 - 55S

F = (19687.5 - 55S) / 30

30((19687.5 - 55S) / 30) + 55S = 19687.5

S = 0 (30%)

F = (19687.5 - 55S) / 30 (55%)

Question 2 (Essay Worth 10 points)
(07.02 MC)
An equation is shown below:
3(2x - 7) = 3
Part A: How many solutions does this equation have? (4 points)
Part B: What are the solutions to this equation? Show your work. (6 points)
ses
GE
22 23
Source
VG
B I
US
IX
SE
E
Styles
Format

Answers

Answer:

Part A: 1 solution

Part B:

3(2x-7)=3

6x-21=3

  +21  +21

6x=24

/6    /6

x=4

X=24

Step-by-step explanation:

First you distribute 3(2x-7)=3 to 6x-21=3. Next you do inverse operations by adding 21 to both sides of the equation to get 6x=24. Finally you divide both sides by 6 to get x=4

Other Questions
The movement of tectonic plates in two locations is described below:Location A: Tectonic plates collide and one plate moves over the other while the other plate is pushed deep into the mantleLocation B: Tectonic plates slide past each otherPLS HELP I WILL GIVE BRAIBLIEST AND 100 POINTSWhich statement is most likely correct? Volcanic eruptions may occur in both locations. There is no chance of earthquakes in the two locations. Mountains may form in Location B but not in Location A. Subduction happens in Location A but not in Location B. NEEEEEDD HELP!CHEMISTRY Sophie has $4.20 worth of dimes and quarters. She has twice as many quarters as dimes. Find the total number of each coin. Can someone plss help me with this and explain this!!! Did you mention that multiplication is a shortcut for repeated addition? (f) A recipe calls for 1 1/2 cups of flour, 3/4 cup of white sugar, and 1/8 cup of brown sugar. (a) How many total cups of ingredients are there? Show your work using a number line.(b) Which of the following mixing bowls can you use to make the recipe: 2-cup, 3-cup, or 4-cup? Justify your answers.Help please! remember show your work on a NUMBER LINE. rewarding 25 points and brainliest. I need help with my math What basic pieces of equipment are needed to get started in archery? which branch of government impeaches/removes the president memona has a 6kg sack of rice and some empty bags. she fills each bag with 475 grams of rice from the sack. how many bags can memona completely fill with rice? If x = 11 and y = 7, what is the value of the following expression?x - 8 + 4y Environment: Polluted Forest Moths ReleasedG1G2G3G4G5 Typica 125 88 83 76 29 Carbonaria 510 735 885 1042 1406 Total 635 823 968 1118 1435 Phenotype Frequency ColorInitial FrequencyFrequency G5 Typica Light 9 Carbonaria Dark Allele Frequency AlleleInitial Allele FrequencyG5 Allele Frequency q d p D Genotype Frequency MothsGenotypeColorMoths ReleasedInitial FrequencyFrequency G5Number of Moths G5 q2 Typica dd Light 2pq Carbonaria Dd Dark p2 Carbonaria DD Dark what is the rotational kinetic energy of the earth? assume the earth is a uniform sphere. data for the earth can be found inside the back cover of the book. HELPPPPPPPPPPPPPPPPPPPPPPPPPPPPPPPPPPPPPPPPPPPPPPPPPPGiven: -3x + y = 1.Write the equation in general form Ax + By + C =0.3x - y = -13x - y + 1 = 0-3x + y - 1 = 0 A tank having a volume of 0.10m' is filled with oxygen at a pressure of 4.0 x 10'Pa and temperature of 47C. Later it is found that, because of a leak, the pressure has dropped to 3.0 x 10'Pa and the temperature has decreased to 27C. Find. (a) the initial mass of oxygen and (b) the mass that has leaked out. Margo has a daily income of $60, which she can spend on coffee or bagels. Each cup of coffee costs $3 and each bagel costs $4. She is currently spending all her income, and she is willing to forgo 1 bagel to get the next cup of coffee to achieve the same level of utility. To maximize her utility, she should:_______.a. increase bagel consumption and decrease coffee consumption b. decrease both consumption of coffee and bagel c. do nothing d. increase coffee consumption and decrease bagel consumption Who answer this right gets 10 pts! Which equation has a solution of r = 1/3 An object is thrown upwards with a velocity of 15 m/s. How long will the ball take to reach its starting point ? Properties of nonmetals includea.high boiling point.b.ability to conduct heat.c.dull appearance.d.none of the above